Please confirm topic selection

Are you sure you want to trigger topic in your Anconeus AI algorithm?

Please confirm action

You are done for today with this topic.

Would you like to start learning session with this topic items scheduled for future?

Review Question - QID 213833

In scope icon M 7 A
QID 213833 (Type "213833" in App Search)
A 57-year-old man presents to the emergency department with chest pain and shortness of breath. It started while the patient was getting his mail and has persisted. The patient has a past medical history of diabetes, hypertension, and morbid obesity. His temperature is 97.9°F (36.6°C), blood pressure is 174/114 mmHg, pulse is 90/min, respirations are 18/min, and oxygen saturation is 98% on room air. Physical exam is notable for an obese, sweaty, and uncomfortable man. An initial ECG demonstrates T wave inversions which are unchanged from a previous ECG. An initial set of troponins is ordered and demonstrates a minor elevation with a second set of troponins trending slightly downward. The patient is given multiple doses of a medication that acts via increasing cGMP. The patient states that his pain has improved. Which of the following changes in Figure A would be expected in this patient after administration of this drug in terms of systemic vascular resistance (SVR), preload, heart rate (HR), and blood flow to the ischemic area of the myocardium (flow)?
  • A
  • A